Is this Integral Convergent or Divergent?

Click For Summary
The integral of (x³+1)/((sinx)^1/2) from 0 to pi/2 is examined for convergence by evaluating the limit as n approaches 0 of the integral from n to pi/2. The integral is considered convergent if this limit exists and is finite. A suggested approach involves comparing the function to a known convergent integral, specifically noting that near 0, (sinx)/x approaches 1, allowing for the approximation of (x³+1)/((sinx)^1/2) to 1/(x^1/2). Since the integral of 1/(x^1/2) converges near 0, it implies that the original integral also converges. The discussion emphasizes the importance of understanding the continuity of the function and the behavior near the endpoints.
Metal
Messages
8
Reaction score
0
How do I know whether this is convergent or divergent:

Integral of (x³+1)/((sinx)^1/2) dx between 0 and pi/2

I know that this integral is convergent if Lim n->0 of Integral of (x³+1)/((sinx)^1/2)) dx between n and pi/2 exists and is not infinite (why is that?). Otherwise its divergent.

So I thought I should find the antiderivative F of (x³+1)/((sinx)^1/2)) and then calculate F(pi/2) - F(n), the problem being that i don't know how to find this F, and I don't think that this is what I'm supposed to do.

Appreciate any help.
 
Physics news on Phys.org
Hint: on the required interval, sinx < x.
 
Metal said:
I know that this integral is convergent if Lim n->0 of Integral of (x³+1)/((sinx)^1/2)) dx between n and pi/2 exists and is not infinite (why is that?). Otherwise its divergent.

Where is this function continuous? That will answer why you only have to worry about the left endpoint.

Metal said:
So I thought I should find the antiderivative F of (x³+1)/((sinx)^1/2)) and then calculate F(pi/2) - F(n), the problem being that i don't know how to find this F, and I don't think that this is what I'm supposed to do.

Try comparing it with an integral you know converges. The sin(x) is the troubling part, but you're near 0 so can you think of something nicer to compare it with?
 
f(x) = (x³+1)/((sinx)^1/2)) ~ g(x) = 1/(x^1/2) near 0 because sinx/x = 1 near 0 (and x³+1/1 too).
Since 1/(x^1/2) is convergent near 0 then f(x) also is.

Is that right?
 
Question: A clock's minute hand has length 4 and its hour hand has length 3. What is the distance between the tips at the moment when it is increasing most rapidly?(Putnam Exam Question) Answer: Making assumption that both the hands moves at constant angular velocities, the answer is ## \sqrt{7} .## But don't you think this assumption is somewhat doubtful and wrong?

Similar threads

  • · Replies 5 ·
Replies
5
Views
2K
  • · Replies 2 ·
Replies
2
Views
2K
  • · Replies 3 ·
Replies
3
Views
1K
  • · Replies 5 ·
Replies
5
Views
2K
Replies
3
Views
2K
  • · Replies 3 ·
Replies
3
Views
7K
  • · Replies 2 ·
Replies
2
Views
2K
  • · Replies 7 ·
Replies
7
Views
2K
  • · Replies 4 ·
Replies
4
Views
1K
  • · Replies 6 ·
Replies
6
Views
2K